LSAT and Law School Admissions Forum

Get expert LSAT preparation and law school admissions advice from PowerScore Test Preparation.

User avatar
 Dave Killoran
PowerScore Staff
  • PowerScore Staff
  • Posts: 5852
  • Joined: Mar 25, 2011
|
#59774
Complete Question Explanation
(The complete setup for this game can be found here: lsat/viewtopic.php?t=26553)

The correct answer choice is (A)

The question stem contains the unusual condition that majors G and H cannot be consecutive, creating a GH not-block. The key to the question is realizing that since G and H are majors, their placement will have a direct effect on the other two majors, F and J, who happen to be in a block configuration. Since the VFJ super-block can only be assigned to three positions—benches 1 and 2, benches 2 and 3, or benches 3 and 4—it makes sense to quickly examine the effect these placements have on G and H. If F and J are assigned to benches 1 and 2, this would force G and H to be consecutive, and if F and J are assigned to benches 3 and 4, this would also force G and H to be consecutive, so the VFJ super-block must be assigned to benches 2 and 3:

pt26_s98_g1_q3.png
Consequently answer choice (A) is proven correct.

Should you find yourself having difficulty with this question, it is interesting to note that the hypothetical produced in question #2 (where G and H were not consecutive) can be used to eliminate answer choices (B), (C), and (D), leaving just answer choices (A) and (E) to attack. This a great example of how using applicable prior work can get you out of difficulty.
You do not have the required permissions to view the files attached to this post.

Get the most out of your LSAT Prep Plus subscription.

Analyze and track your performance with our Testing and Analytics Package.